Look at image that is attached

Look At Image That Is Attached

Answers

Answer 1

Answer: Linear

Step-by-step explanation:

I can tell from the table because there is a difference of 4 between each y value.

I dont know what the other blanks will be because i cant see the options. But it shouldnt be too hard to fill out


Related Questions

Draw a net to represent the following 3d shape.
Don't forget your lebel where the 6cm and 4cm are on the net.

Answers

Answer: Discover how a 3D solid shape can be made up from a 2D net. Understand how nets are formed with examples of common 3D polygons and prisms.

Step-by-step explanation:

Hurry up please
State the domain and range and tell if the graph is a function yes or no
What’s the domain and range?

Answers

The graph is a function: yes

The domain of this function is: x ≥ -3.

The range of this function: all real numbers.

What is a domain?

In Mathematics and Geometry, a domain is the set of all real numbers for which a particular function is defined.

Additionally, the vertical extent of any graph of a function represents all range values and they are always read and written from smaller to larger numerical values, and from the bottom of the graph to the top.

By critically observing the graph shown in the image attached above, we can reasonably and logically deduce the following domain and range:

Domain = {-3, ∞}, x ≥ -3, or -3 ≤ x ≤ ∞.

Range = {-∞, ∞} or all real numbers.

Read more on domain here: brainly.com/question/17440903

#SPJ1

Answers for these test question, thank you

Answers

The missing lengths of the following two triangles:

Case 1: x = 50 km

Case 2: x = 40 km

Case 3: Obtuse

Case 4: Acute

Case 5: Right

Case 6: m = 16, n = 8√3

Case 7: x = 3√2, y = 3

How to find the length of missing sides in a triangle

In this problem we must determine all missing lenghts in a triangle, this can be done by law of cosine:

x² = a² + b² - 2 · a · b · cos X

Where X is the angle opposite to side x.

Now we proceed to determine the missing lengths of the following two triangles:

Case 1

x = √[(14 km)² + (48 km)² - 2 · (14 km) · (48 km) · cos 90°]

x = 50 km

Case 2

x = √[(24 km)² + (32 km)² - 2 · (24 km) · (32 km) · cos 90°]

x = 40 km

In addition, we can determine if any triangle is acute, obtuse and right also by law of cosine:

cos X = - (x² - a² - b²) / (2 · a · b)

Case 3

cos X = - [(18 in)² - (12 in)² - (9 in)²] / [2 · (12 in) · (9 in)]

cos X = - 0.458 (Obtuse)

Case 4

cos X = - [(15 ft)² - (12 in)² - (14 in)²] / [2 · (12 in) · (14 in)]

cos X = 0.342 (Acute)

Case 5

cos X = - [(5 ft)² - (3 in)² - (4 in)²] / [2 · (3 in) · (4 in)]

cos X = 0 (Right)

Finaly, we determine the missing lengths of two right triangles by trigonometric functions:

sin θ = y / r

cos θ = x / r

tan θ = y / x

Now we find the missing lengths:

Case 6

m = 8 / cos 60°

m = 16

n = 8 · tan 60°

n = 8√3

Case 7

x = 3 / sin 45°

x = 3√2

y = 3 / tan 45°

y = 3

To learn more on law of cosine: https://brainly.com/question/30766161

#SPJ1

Which type of relationship is indicated by the residual plot? Non-linear relationship. Linear relationship Exponential relationship

Answers

The type of relationship that is indicated by the residual plot is given as follows:

Non-linear relationship.

How to classify the functions?

A function is classified as exponential if when the input variable is changed by one, the output variable is multiplied by a constant.

A function is classified as linear if when the input variable is changed by one, the output variable is increased/decreased by a constant.

For this problem, we have an alternative pattern, where the function neither multiplied nor added/subtracted by a constant value when the input is increased by one, hence we merely classify the relationship as a non-linear relationship.

More can be learned about exponential functions at brainly.com/question/2456547

#SPJ1

A line has a slope of 3 and y-intercept of -4. Write its equation in slope- intercept form.​

Answers

Answer:

y=mx + b

Step-by-step explanation:

y=3x-4

3 is the slope and -4 is the y intercept

Two forces act at a point in the plane. the angles between the two forces are given. fidn the magnitude of the resultant force

Answers

The resultant between the two vectors is 21.17 lb

What is parallelogram law of vectors?

Parallelogram law of vectors says that if two vectors acts on a point, the resultant of this vectors can be represented by the diagonal from the point of intersection.

R² = x²+y2+2xycos (tetha)

where x and y are the vectors

When x = 20.6 and y = 17.4

R² = 20.6²+17.4²+2(20.6)(17.4)cos 112.9

R² = 727.12+716.88cos 112.9

R² = 727.12 -278.95

R² = 448.17

R = √448.17

R = 21.17 lb

Therefore the resultant is 21.17lb

learn more about parallelogram law of vectors from

https://brainly.com/question/23867486

#SPJ1

a principal data about the distance, in miles that his teachers and bus drivers live from the school, the box plots below show these data. based on the box plots, which statement is true?

Answers

Option C is the true statement from the data that has been gathered and shown in the image

How to get the true statement

The interquartile range exemplifies the "dispersion" or width of a set [1] by determining the difference between the top quartile (the 25% highest) and lower quartile (the 25% lowest). In reference to the provided picture:

- The bus drivers' interquartile range is 10, represented by subtracting their distance's lowest point (10) from the highest (20).

20 - 10 = 20

- Similarly, the teachers' interquartile range is 15, which stems from finding the difference between their lowest distance (15) and highest distance (30).

30 - 15 = 15

Therefore, comparing both ranges reflects that the bus drivers have an interquartile range of distances that is 5 miles smaller than the one for the teachers. Consequently, we opt for option C.

Read more on interquartile range here: https://brainly.com/question/4102829

#SPJ1

Solve for a.
60°
a
60°
a = [? ]°

Answers

Final answer:

The value of a in the equation 60°a = [? ]° is dependent on the value of [? ].

Explanation:

To solve for a in the equation 60°a = [? ]°, we need to isolate a on one side of the equation. We can do this by dividing both sides of the equation by 60°:

60°a / 60° = [? ]° / 60°

This simplifies to:

a = [? ]° / 60°

Since we don't have the value of [? ]°, we cannot determine the exact value of a. However, we can simplify the expression by canceling out the ° units:

a = [? ] / 60

Therefore, the value of a is dependent on the value of [? ].

Learn more about solving for a in an equation involving angles here:

https://brainly.com/question/29261700

#SPJ14

Help please 3rd times a charm

Answers

The amount that you would save in annual fuel expenses would be $9, 333. 32

The amount that you would have saved over the five years would be $52, 854.63

How to find the fuel savings ?

The amount saved in fuel when using the hybrid is:

= Fuel usage by SUV - Fuel usage by Hybrid

= ( 30, 000 / 9 x 4 ) - ( 30, 000 / 30 x 4 )

= $9, 333.32

If you saved this amount as an annuity, it would come out to:

=  ( 9, 333.32 / 12 ) x  [ ( 1 + 0. 004333 ) ^ ( 12 x 5 ) - 1 ] / 0. 004333

= 777. 78 x 0. 2957 / 0. 004333

= $ 52, 854.63

Find out more fuel savings at https://brainly.com/question/17647831

#SPJ1

Fill in the blanks using the available answers


I believe the first part is 2.2. But lost on the fill in the blanks. Help please.

Answers

Note that the estimated population mean is 2.2

The mean absolute deviation is 1

The average distance each data value is from the mean is 1 hours per night

The distribution has _Low_ variability

How did we reach the above conclusions?

Where the data graphed is:

Hours Per Night       Students

0.5                                       0

1                                            2

1.5                                           3

2                                              4

2.5                                        2

3                                             2

3.5                                          1

4                                             1


The estimate of the population mean is

0.5 (0 ) + 1 ( 2) +  1.5( 3) + 2(4) + 2.5(2) + 3( 2 ) + 3.5( 1) + 4( 1) =3 3

⇒ 33 / 15

population Mean = 2.2



To find the Mean Absolute Deviation:

0.5 - 2.2 = - 1.7

1 -  2.2 = -1 .2

1.5 - 2.2   = - 0.7

2 -  2.2  = - 0.2

2.5   - 2.2  =0.3

3  -  2.2 = 0.8

3.5 - 2.2 = 1.3

4 - 2.2 =   1.8

The absolute deviations from the sample mean are

1.7 ,  1.2, 0.7, 0.2 , 0.3, 0.8 , 1.3,1.8

Thus

(1.7 + 1.2 + 0.7 + 0.2+0.3 + 0.8 + 1.3 +1.8) / 8 = 8/8 = 1

Therefore, the M.A. D is 1 hours per nigh


For average distance each data value is from the mean...

|(-1.7)| + |( -1.2) | + | ( -0.7)| + |( -0.2 | + | 0.3| + |  0.8| + | 1.3 | + | 1.8 | = 8

8/ 8 = 1


So average distance of each data value form the mean is 1 hours per night.


We discovered that the M.A. D in this scenario is 1hr/ night. We may claim that the distribution has low variability because this number is quite tiny.

Learn more about Mean Absolute Deviation:
https://brainly.com/question/10528201
#SPJ1

The length of a picture frame is 7 inches more than the width. For what values of x is the perimeter of the picture frame greater than 150 inches?

Answers

Let x be the width of the picture frame. Then, the length of the picture frame is x + 7. The perimeter of the picture frame is 2(x + x + 7) = 4x + 14. To find the values of x for which the perimeter of the picture frame is greater than 150 inches, we solve the inequality 4x + 14 > 150. Subtracting 14 from both sides gives 4x > 136. Dividing both sides by 4 gives x > 34. Therefore, the values of x for which the perimeter of the picture frame is greater than 150 inches are x > 34.

a rectangular prism has 312 cubes. The cubes have an edge length of 1/5 cm. What is the volume of this rectangular prism?

Answers

Answer:

[tex] {( \frac{1}{5}) }^{3} (312) = \frac{312}{125} = 2.496[/tex]

The volume of this rectangular prism is 2.496 cubic centimeters.

The volume of the rectangular prism is 2.496 cubic centimeters.

If the rectangular prism has 312 cubes with an edge length of 1/5 cm, we may calculate the total volume by multiplying the number of cubes by the volume of each cube.

V = x3 gives the volume of a cube with edge length x. Because the edge length, in this case, is 1/5 cm, the volume of each cube is (1/5)3 = 1/125 cm3.

We multiply the number of cubes by the volume of each cube to determine the volume of the rectangular prism:

312 cubes (1/125 cm3/cube) = 2.496cm3 volume

Learn more about cubes:

https://brainly.com/question/28134860

The annual profits for a company are given in the following table, where x represents the number of years since 2012, and y represents the profit in thousands of dollars. Write the linear regression equation that represents this set of data, rounding all coefficients to the nearest tenth. Using this equation, find the projected profit (in thousands of dollars) for 2022, rounded to the nearest thousand dollars.

Answers

The linear regression equation is y = 22.6x + 52.33

The profit would reach 234 thousand dollars during the calendar year of 2020.

How to calculate the value

Replacing the means, the intercept a is given by:

108.8 = 22.6(2.5) + a.

a = 108.8 - 22.6(2.5)

a = 52.33

Therefore, the linear regression equation is y = 22.6x + 52.33

The profit would reach 234 thousand dollars during the calendar year of 2020.

Learn more about regression on

https://brainly.com/question/21826199

#SPJ1

Answer:

Step-by-step explanation:

Which of the following scatterplots shows a correlation affected by an influential point?

Answers

Answer:

The correlation coefficient of a data set can be significantly altered by removing an influential point from the data set.

Step-by-step explanation:

Please give the brainliest, really appreciated. Thank you

Suppose we wish to factor 13b2 + 23b - 8 by grouping. What is the key number, ac?

Answers

The key number, ac, is -104.

What is key number ?

Key number is the product of the coefficients of the first and last terms. The right binomial factors for a quadratic expression can be determined with its assistance.

We must multiply the coefficient of the quadratic term by the constant term to obtain the key value, ac, which results in:

ac = 13(-8) = -104

Therefore, The key number, ac, is -104.

Learn more about key number here : brainly.com/question/29574610

#SPJ1

Help me please please please

Answers

Answer:

C you are so welcome I hope it helped you

José has a wedge-shaped piece of wood as shown in the diagram. José plans to paint the piece
needs.
3 in.
4 in.
5 in.
8 in.

Answers

Answer:

E. 108 in.²

Step-by-step explanation:

The piece of wood has the shape of a triangular prism.

SA = lateral area + area of the bases

SA = perimeter × height + 2 × bh/2

SA = (5 + 3 + 4) in. × 8 in. + 3 in. × 4 in.

SA = 108 in.²

when its comes to interest,most CDs will allow which of the following options?
a. Unlimited withdraws of interest
b. Periodic interest payout
c. loans against principal
d. Withdrawal of principal

Answers

When it comes to interest, most Certificates of Deposit (CDs) will allow periodic interest payouts. So, correct option is B.

A CD is a type of savings account that allows you to earn a fixed interest rate on your deposit for a specific period of time, called the term. Typically, the longer the term of the CD, the higher the interest rate offered.

However, during the term of the CD, the funds are locked in, meaning that you cannot withdraw the principal without incurring a penalty.

While some CDs may allow for unlimited withdrawals of interest, this is not common, and may still come with restrictions or penalties. Loans against principal are also not typically allowed with CDs, as the funds are meant to be held for a set term.

Therefore, the most common option available for CD holders is to receive periodic interest payouts, which can be monthly, quarterly, or annually, depending on the terms of the CD. This allows the CD holder to earn interest on their deposit while still receiving some income during the term of the CD.

So, correct option is B.

To learn more about interest click on,

https://brainly.com/question/12456231

#SPJ1

Manuel bought a car for 65% of the original price of $7,000. How much did he pay for the car

Answers

Manuel bought a car for 65% (percentage) of the original price of $7,000, he must have paid $4,550 for the car.

What is the percentage?

The percentage is a number or ratio that represents a fraction of a whole number or the sum of ratios.

The percentage is computed by dividing one number by another and multiplying the result by 100.

The original price of the car = $7,000

The discount factor = 65% (1 - 35%) or 0.65

The discounted price = $4,550 ($7,000 x 0.65)

Thus, by implication, for receiving a discount of 35% (100% - 65%), Manuel paid $4,550 for the car.

Learn more about discounts at https://brainly.com/question/12965533.

#SPJ1

Pls tell me just the answer

Answers

The domain of the function g(x) is given as follows:

B. All values of x such that x ≥ 0.

How to obtain the domain of the function?

The domain of a function is the set of all the possible input values that can be assumed by the function.

The function for this problem is defined as follows:

[tex]g(x) = \sqrt{8x}[/tex]

The square root function assumes values that are greater than or equals to zero, hence the domain of the function is obtained as follows:

8x ≥ 0.

x ≥ 0.

More can be learned about the domain of a function at https://brainly.com/question/10687170

#SPJ1

Determine each lengths in right triangle ABC.
BD———>
AB———>

Answers

The missing parts of the triangle are

BD = 8

AD = 8 sqrt(2)

How to find the missing parts

The missing part BD of the figure is solved using similar triangles

The expression is as follows

h / 8 = 8 / h

h^2 = 8 x 8

h^2 = 64

h sqrt = (64)

h = 8

Solving for AB we use Pythagoras theorem

AB^2 = h^2 + 8^2

AB^2 = 8^2 + 8^2

AB = sqrt(64 + 64)

AB = sqrt (128)

AB = 8 sqrt (2)

Learn more about Pythagoras theorem at

https://brainly.com/question/231802

#SPJ1

What is the x of a 68⁰? I’m in 8th grade learning about angels and basically adding them .

Answers

The complement and the supplement of an angle of 68º are given as follows:

Complement: 22º.Supplement: 112º.

How to obtain the complement of an angle?

When two angles are complementary, the sum of the measures of the angles is of 90º, hence the complement of an angle is obtained subtracting 90º by the angle measure.When two angles are supplementary, the sum of the measures of the angles is of 180º, hence the supplement of an angle is obtained subtracting 180º by the angle measure.

Hence the complement and the supplement of an angle of 68º are obtained as follows:

Complement: 90 - 68 = 22º.Supplement: 180 - 68 = 112º.

More can be learned about complementary and supplementary angles at https://brainly.com/question/2046046

#SPJ1

La empresa clarisse dedicada al rubro de telefónica, está ampliando su cobertura, debido a esto está colocando postes, donde la séptima parte de un poste está enterrada, y sobresale del suelo 25 metros. Calcular la altura del poste aproximado a los centímetros.

Answers

The approximate height of the pole in centimeters is 17,500 centimeters.

We have,

Let's start by calculating the total length of the pole, which is buried plus the part that protrudes from the ground.

We know that the part that protrudes is 25 meters and that it represents one-seventh of the total length.

25 = L/7

where L is the total length of the pole.

We can solve for L by multiplying both sides of the equation by 7:

L = 7 x 25

  = 175 meters

Now we need to convert this length to centimeters.

Since 1 meter is equal to 100 centimeters,

175 meters x 100

= 17,500 centimeters

Therefore,

The approximate height of the pole in centimeters is 17,500 centimeters.

Learn more about unit conversion here:

https://brainly.com/question/13899873

#SPJ1

The complete question.

The Clarisse company, dedicated to the telephone business, is expanding its coverage, due to this, it is placing poles, where the seventh part of a pole is buried, and protrudes 25 meters from the ground. Calculate the approximate height of the pole in centimeters.

The chief physician at a hospital wants to analyze the amount of time it takes two doctors to complete a particular surgical procedure. A sample of 35 of these procedures performed by Dr. McCoy were completed in a mean time of 60.3 minutes with a standard deviation of 4.3 minutes. A sample of 41 these procedures performed by Dr. Turk were completed in a mean time of 61.2 minutes with a standard deviation of 4.7 minutes. A claim is made that the mean time for Dr. McCoy (
ц1) is less than the mean time for Dr. Turk ц2
).

For each part below, enter only a numeric value in the answer box. For example, do not type "z =" or "t =" before your answers. Round each of your answers to 3 places after the decimal point.

(a) Calculate the value of the test statistic used in this test.

Test statistic's value =

(b) Use your calculator to find the P-value of this test.

P-value =

Submit QuestionQuestion 9

Answers

The value of the test statistic used in this test is t = -2.168 and the P-value of this test is P-value is 0.034

To test whether the mean time for Dr. McCoy is different from the mean time for Dr. Turk, we can use a two-sample t-test with unequal variances. The test statistic for this test is given by:

t = (X₁ - X₂) / √(s₁²/n₁ + s₂²/n₂)

where X₁ and X₂ are the sample means, s₁ and s₂ are the sample standard deviations, and n₁ and n₂ are the sample sizes.

Plugging in the given values, we get:

t = (78 - 78.7) / √(2.3²/38 + 4.1²/33)

= -2.168

Therefore, the value of the test statistic used in this test is t = -2.168.

b) To find the P-value of this test, we need to use a t-distribution with degrees of freedom given by:

df = (s₁²/n₁+ s₂²/n₂)² / [ (s₁²/n₁)²/(n₁-1) + (s₂²/n₂)²/(n₂-1) ]

Plugging in the given values, we get:

df = (2.3²/38 + 4.1²/33)² / [ (2.3²/38)²/37 + (4.1²/33)²/32 ] ≈ 65.385

Using a t-distribution table or a calculator with t-distribution function, we find that the P-value for a two-tailed test with t = -2.168 and df = 65.385 is approximately 0.034.

Therefore, the P-value of this test is P-value = 0.034

To learn more on Statistics click:

https://brainly.com/question/30218856

#SPJ1

Solve the following for θ, in radians, where 0≤θ<2π.
−7cos2(θ)+3cos(θ)+7=0
Select all that apply:

1.77
0.48
1.4
3.77
2.99
2.51

Answers

Answer:3.77

2.51 are correct

Step-by-step explanation:We can solve this quadratic equation in cos(θ) by using the substitution u = cos(θ):

-7u^2 + 3u + 7 = 0

Multiplying both sides by -1, we get:

7u^2 - 3u - 7 = 0

We can use the quadratic formula to solve for u:

u = (-b ± sqrt(b^2 - 4ac)) / 2a

where a = 7, b = -3, and c = -7. Substituting these values, we get:

u = (3 ± sqrt(9 + 196)) / 14

u = (3 ± 5sqrt(5)) / 14

Therefore, either:

As seen in the diagram below, Arun is building a walkway with a width of x feet to go around a swimming pool that measures 15 feet by 7 feet. If the total area of the pool and the walkway will be 713 square feet, how wide should the walkway be?​

Answers

The walkway around the swimming pool should be 8 feet wide.

Calculating the width of the walkway

From the question, we are given:

Length of the pool (l) = 15 feet

Width of the pool (w) = 7 feet

width of walkway (X) = X feet

Total Area =  Area of pool and walkway = 713 square feet

Let's derive equation for the pool and walkaway combined:

length of the pool and walkway = 15 + 2X [walkway is on both sides]

width of the pool and walkway = 7 + 2X

Total Area = (length x width) of the pool and walkway

713 = (15 + 2X) x (7 + 2X)

713 = 15(7 + 2X) + 2X(7 + 2X)

713 = 105 + 30X + 14X + 4X²

713 = 105 + 44X + 4X²

Rearrange the equation

4X² + 44X + 105 = 713

Collect like terms and express in a proper quadratic equation

4X² + 44X - 608 = 0

Dividing both sides by 4, we get

X² + 11X - 152 = 0

Solve this equation quadratically

X =  [tex]\frac{-b \± \sqrt{b^{2} - 4ac}}{2a}[/tex]

where

a = 1

b = 11

c = -152

Plug in the value into the equation

X = [tex]\frac{-11 \± \sqrt{11^{2} - 4(1)(-152)}}{2(1)}[/tex]

X = [tex]\frac{-11 \± \sqrt{121 + 608}}{2(1)}[/tex]

X = [tex]\frac{-11 \± \sqrt{729}}{2}[/tex]

X = [tex]\frac{-11 \± 27}{2}[/tex]

X = [tex]\frac{-11 + 27}{2}[/tex] or [tex]\frac{-11 - 27}{2}[/tex]

X = [tex]\frac{16}{2}[/tex] or [tex]\frac{-38}{2}[/tex]

X = 8 or -19

But since the width of the walkway cannot be negative, the only valid solution is therefore:

X = 8

Learn more about width here:

https://brainly.com/question/28107004

#SPJ1

Round to the nearest tenth, then find the sum. 35.26 + 8.32 + 6.78 i will give 12 points pleas help me in in a test

Answers

Answer:

Step-by-step explanation:

35.26    rounded to  35.3

8.32   rounded to        8.3

6.78    rounded to       6.8

35.3 + 8.3 = 43.6

43.6 + 6.8 = 50.4

The answer is 50.4.

39.26 rounds to 35.3

8.32 rounds to 8.3

6.78 rounds to 6.8

When you add them together, you get 50.4.

if $5700 is invested in a savings account for which interest is compounded annually, and if the $5700 turns into $6100 in 12 years, what is the interest rate of the savings account?

Answers

Answer:

We can use the formula for compound interest:

A = P(1 + r/n)^(nt)

where A is the ending amount, P is the principal (starting amount), r is the interest rate (as a decimal), n is the number of times the interest is compounded per year, and t is the time in years.

We know P = $5700, A = $6100, n = 1 (since the interest is compounded annually), and t = 12. We can solve for r:

$6100 = $5700(1 + r/1)^(1*12)

$6100/$5700 = (1 + r)^12

1.0702 = (1 + r)^12

log(1.0702) = log[(1 + r)^12]

0.0291 = 12log(1 + r)

0.00243 = log(1 + r)

10^(0.00243) = 1 + r

r = 0.0059 or 0.59%

Therefore, the interest rate of the savings account is 0.59%.

X
-2
-1
01
2
3
-2
0
7
What is the rate of change for the interval between 0
and 2 for the quadratic equation as f(x)=2x²+x-3
represented in the table?
O
5
10

Answers

The rate of change for the interval between 0 and 2 for the quadratic equation is 10.

What is the change in interval of the quadratic equation?

A quadratic equation is a type of polynomial equation of the second degree, which means it has one or more terms that are raised to the power of two.

The general form is;

ax² + bx + c = 0

Where;

x is the variablea, b, and c are constants

The rate of change for the interval between 0 and 2 for the quadratic equation is calculated as;

f(2) - f(0) = 7 - (-3)

             = 7 + 3

             = 10

Learn more about quadratic equation here: https://brainly.com/question/1214333

#SPJ1

6x3 − 4x2 + 11, x + 3

Answers

The quotient of the long division of the polynomial (6x³ - 4x² + 11)/(x+3)  is 6x² - 22x + 66.

What are the quotient of the polynomial?

The quotient of the polynomial divided by a factor of x + 3 is determined by applying long division method as shown below;

6x³ - 4x² + 11 ÷  x + 3

                   6x² - 22x + 66

              ------------------------

  x + 3   √ 6x³ - 4x² + 11

               - (6x³ + 18x²)

             -------------------------

                         -22x² + 11

                    -   (-22x² - 66x)

                -----------------------------

                                  66x + 11

                            -  (66x + 198)

                 ---------------------------------

                                      -187

Thus, the quotient of the long division of the polynomial is obtained as   6x² - 22x + 66.

Learn more about long division here: https://brainly.com/question/25289437

#SPJ1    

Find the quotient of the polynomial using long division

6x3 − 4x2 + 11/x + 3

Other Questions
The brain is a solid organ that lacks cavities. true/false On-jar coupons, In-jar coupons, newspaper coupons, electronic coupons, who cares they all serve the same purpose. Select one:TrueFalse The "NURTURE" part of the nature vs. nurture controversy is best described as: help me please deadline tomorrow please Caves are formed when rainwater mixes with the carbon dioxide in the air and Cullumber Company sells its product for $25 per unit. During 2022, it produced 24000 units and sold 15000 units (there was no beginning inventory). Costs per unit are direct materials 55, direct labour $4, and variable overhead $3. Fixed costs are: $288000 manufacturing overhead and $48000 selling and administrative expenses. Under absorption costing what amount of fived overhead is deferred to a future period? O $288000 O S180000 $108000 $99000 Discuss two ways in which the strength of a skeletal muscle contraction can be changed A 3.1-kg box is sliding along a frictionless horizontal surface with a speed of 1.8 m/s when it encounters a spring. (a) Determine the force constant of the spring, if the box compresses the spring 5.3 cm before coming to rest. N/m (b) Determine the initial speed the box would need in order to compress the spring by 1.6 cm. m/s PLEASE HELP ITS URGENT I INCLUDED THE GRAPH AND WROTE THE PROBLEM DOWN ITS THE IMAGE I HAVE ATTACHED!!! dr j assigns you and a student you do not know a joint project. assume that you and your partner are both interested in maximizing your grade, but you are both very busy and get more happiness if you can get a good grade with less work. since you do not know the student, making a credible commitment to how much you are going to work is impossible. a reproduction of a sculpture is made at a scale of 1:15 the reproduction is 13cm tall what is the height of the original sculpture in centimeters Square ABCD with vertices A(-7,5) B(-4,7) C(-2,4) and D(-5,2) 90 counterclockwise PLEASE ANSWER ASAPQuestion 19 (Multiple Choice Worth 1 points)(The Enlightenment and Revolution: The Scientific Revolution LC)Which of the following was an invention from the Scientific Revolution?O LightbulbGunpowderMicroscopeTelevision The given planes intersect in a line. Find parametric equations for the line of intersection. [Hint: The line of intersection consists of all points (x, y, z) that satisfy both equations. Solve the system and designate the unconstrained variable as t .]x + 2y + z = 1, 2x+5y + 32 = 4 Answer the following questions about options written on a dividend paying stock. Suppose that S = 50, K = 45, R = 1.03, u = 1.2, d = .8, and that the stock pays dividends at a rate & such that $\theta=e^{\delta h}=1.06$. i. What is the value of a European call two periods from expiration? ii. What is the value of a European put two periods from expiration? iii. What is the value of an American call two periods from expiration? iv. What is the value of an American put two periods from expiration? C++ please ... thank youDefine a structure named StockItem with two string fields, supplier and productName and one int field, catalogNumber. Then define a structure named Customer with string fields name , streetAddress, city, postalCode, phone. Assume that structures named Date andMoney have already been defined (representing a date and a monetary amount respectively.Finally, define a structure named Purchase that has these fields: buyer of type Customer, itemSold of type StockItem, dateOfSale of type Date, paid of type Money, returnable of type bool.142. Messing Company has an agreement with a third-party credit card company, which calls for cash to be received immediately upon deposit of customers' credit card sales receipts. The credit card company receives 3.5 percent of card sales as its fee. Messing has $4.000 in credit card sales on January 1 Prepare the January 1 journal entry for Messing Company by selecting the account names from the drop-down menus and entering the dollar amounts in the debitor credit columns. How many tons are equal to 36,000 pounds?O 1,800 tonsO 180 tonsO 18 tons08 tons What are fifth step in a blow fly life cycle? lack of responsiveness and failure to protect are examples of which of the following types of prison abuse identified by bomse?Malicious or purposeful abuseNegligent abuseSystemic or Budgetary Directions: There are 11 questions in 5 pages. No credit will be given without sufficient work 1. Let Z be a standard normally distributed random variable. Find: a. P(Z S 2.32) b. P(Z 2-1.56) c. P(-1.43 SZ 52.47) d. Find : so that P(-:* SZS :) 0.99